Những câu hỏi liên quan
Vũ Thu Mai
Xem chi tiết
trần thành đạt
2 tháng 1 2018 lúc 16:59

bài 1 a, hình như có thêm đk là a+b+c=3

Bình luận (0)
trần thành đạt
2 tháng 1 2018 lúc 17:14

Bài 4 nha

Áp dụng BĐT cô si ta có

\(\frac{1}{x^2}+x+x\ge3\sqrt[3]{\frac{1}{x^2}.x.x}=3.\)

Tương tự với y . \(A\ge6\)dấu = xảy ra khi x=y=1

Bình luận (0)
trần thành đạt
2 tháng 1 2018 lúc 20:11

câu 1 mk bị lộn nhưng đáng ra  ca^2 thành c^2a  mới đúng

Bình luận (0)
nub
Xem chi tiết
tth_new
31 tháng 5 2020 lúc 18:37

Bài 1. Ta có: \(a\left(a+2\right)\left(a-1\right)^2\ge0\therefore\frac{1}{4a^2-2a+1}\ge\frac{1}{a^4+a^2+1}\)

Thiết lập tương tự 2 BĐT còn lại và cộng theo vế rồi dùng Vasc (https://olm.vn/hoi-dap/detail/255345443802.html)

Bài 5: Bất đẳng thức này đúng với mọi a, b, c là các số thực. Chứng minh:

Quy đồng và chú ý các mẫu thức đều không âm, ta cần chứng minh:

\(\frac{1}{2}\left(a^2+b^2+c^2-ab-bc-ca\right)\Sigma\left[\left(a^2+b^2\right)+2c^2\right]\left(a-b\right)^2\ge0\)

Đây là điều hiển nhiên.

Bình luận (0)
 Khách vãng lai đã xóa
emily
Xem chi tiết
Ngô Ngọc Quỳnh Mai
1 tháng 12 2016 lúc 11:22

1) = 3n(32+1) - 2n(22+1)

2)A=m.n.p

\(\frac{m^2}{\frac{2^2}{5^2}}=\frac{n^2}{\frac{3^2}{4^2}}=\frac{p^2}{\frac{1^2}{6^2}}=\frac{m^2+n^2+p^2}{\frac{2^2}{5^2}+\frac{3^2}{4^2}+\frac{1^2}{6^2}}\)

3) \(\frac{a^2}{\text{\text{c}^2}}=\frac{\text{c}^2}{b^2}=\frac{a^2+\text{c}^2}{b^2+\text{c}^2}\)\(\frac{a^2}{\text{c}^2}=\frac{\text{c}^2}{b^2}=\frac{a^2+\text{c}^2}{\text{c}^2+b^2}\)

mà ab=c2

suy ra đpcm

Bình luận (0)
Haa My
Xem chi tiết
Hồng Phúc
7 tháng 10 2020 lúc 18:37

1.

\(10x=|x+\dfrac{1}{10}|+|x+\dfrac{2}{10}|+...+|x+\dfrac{9}{10}| \ge 0\)

\(\Rightarrow x\ge0\)

\(pt\Leftrightarrow x+\frac{1}{10}+x+\frac{2}{10}+...+x+\frac{9}{10}=10x\)

\(\Leftrightarrow x=\frac{1}{10}+\frac{2}{10}+...+\frac{9}{10}=\frac{9}{2}\)

\(\Rightarrow x=\frac{9}{2}\)

Bình luận (0)
 Khách vãng lai đã xóa
Hồng Phúc
7 tháng 10 2020 lúc 19:07

4.

Áp dụng tính chất dãy tỉ số bằng nhau

\(\frac{a}{b+3c}=\frac{b}{c+3a}=\frac{c}{a+3b}=\frac{a+b+c}{4\left(a+b+c\right)}=\frac{1}{4}\)

\(\Rightarrow\left\{{}\begin{matrix}4a=b+3c\left(1\right)\\4b=c+3a\left(2\right)\\4c=a+3b\left(3\right)\end{matrix}\right.\)

Từ \(\left(1\right);\left(2\right)\Rightarrow4a=b+3\left(4b-3a\right)\)

\(\Rightarrow12a=12b\Rightarrow a=b\left(4\right)\)

Từ \(\left(1\right);\left(3\right)\Rightarrow4c=a+3\left(4a-3c\right)\)

\(\Rightarrow12a=12c\Rightarrow a=c\left(5\right)\)

Từ \(\left(4\right);\left(5\right)\Rightarrow a=b=c\left(đpcm\right)\)

Bình luận (0)
 Khách vãng lai đã xóa
Hồng Phúc
7 tháng 10 2020 lúc 19:22

a, \(M=\frac{1}{4}+\frac{1}{4^2}+...+\frac{1}{4^n}\)

\(\Rightarrow4M=1+\frac{1}{4}+...+\frac{1}{4^{n-1}}\)

\(\Rightarrow3M=1-\frac{1}{4^n}< 1\Rightarrow M< \frac{1}{3}\left(đpcm\right)\)

b, Lập luận tương tự câu a

\(M=\frac{1}{4}+\frac{1}{4^2}+...+\frac{1}{4^n}\)

\(\Rightarrow4M=1+\frac{1}{4}+...+\frac{1}{4^{n-1}}\)

\(\Rightarrow3M=1-\frac{1}{4^n}< 1+\frac{1}{3}=\frac{4}{3}\Rightarrow M< \frac{4}{9}\left(đpcm\right)\)

Bình luận (0)
 Khách vãng lai đã xóa
Nguyễn Bùi Đại Hiệp
Xem chi tiết
bach nhac lam
19 tháng 5 2020 lúc 23:11

Đề: \(\frac{1}{\sqrt{a^4-a^3+ab+2}}+\frac{1}{\sqrt{b^4-b^3+bc+2}}+\frac{1}{\sqrt{c^4-c^3+ca+2}}\le\sqrt{3}\) ???

*Ta chứng minh : \(x^4-x^3+2\ge x+1\forall x>0\)

\(\Leftrightarrow x^4-x^3-x+1\ge0\Leftrightarrow\left(x-1\right)^2\left(x^2+x+1\right)\ge0\) ( đúng )

Do đó: \(VT\le\frac{1}{\sqrt{ab+a+1}}+\frac{1}{\sqrt{bc+b+1}}+\frac{1}{\sqrt{ca+c+1}}\) \(\le\sqrt{3\left(\frac{1}{ab+a+1}+\frac{1}{bc+b+1}+\frac{1}{ca+c+1}\right)}=\sqrt{3}\)

Dấu "=" \(\Leftrightarrow a=b=c=1\)

Bình luận (0)
Ngô Hải Hà
Xem chi tiết
Thắng Nguyễn
19 tháng 4 2016 lúc 18:48

câu 2:đặt B=1/1*2+1/2*3+...+1/2007*2008

ta có:\(A=3\left(\frac{1}{2^2}+\frac{1}{3^2}+...+\frac{1}{2008^2}\right)\)

\(\frac{A}{3}=\frac{1}{2^2}+\frac{1}{3^2}+...+\frac{1}{2008^2}

Bình luận (0)
Thắng Nguyễn
19 tháng 4 2016 lúc 18:52

câu 2:đặt B=1/1*2+1/2*3+...+1/2007*2008

\(A=3\left(\frac{1}{2^2}+\frac{1}{3^2}+...+\frac{1}{2008^2}\right)\)

\(\frac{A}{3}=\frac{1}{2^2}+\frac{1}{3^2}+...+\frac{1}{2008^2}\)\( (1)

Bình luận (0)
Nguyễn Minh Đăng
Xem chi tiết
Kiệt Nguyễn
20 tháng 8 2020 lúc 9:19

Bất đẳng thức cần chứng minh tương đương: \(\frac{1}{a^4\left(b+1\right)\left(c+1\right)}+\frac{1}{b^4\left(c+1\right)\left(a+1\right)}+\frac{1}{c^4\left(a+1\right)\left(b+1\right)}\ge\frac{3}{4}\)

Đặt \(a=\frac{1}{x};b=\frac{1}{y};c=\frac{1}{z}\)thì \(\hept{\begin{cases}x,y,z>0\\xyz=1\end{cases}}\)và ta đưa BĐT cần chứng minh về dạng \(\frac{x^3}{\left(y+1\right)\left(z+1\right)}+\frac{y^3}{\left(z+1\right)\left(x+1\right)}+\frac{z^3}{\left(x+1\right)\left(y+1\right)}\ge\frac{3}{4}\)

Áp dụng BĐT AM - GM, ta được:\(\frac{x^3}{\left(y+1\right)\left(z+1\right)}+\frac{y+1}{8}+\frac{z+1}{8}\ge\frac{3}{4}x\)

Tương tự: \(\frac{y^3}{\left(z+1\right)\left(x+1\right)}+\frac{z+1}{8}+\frac{x+1}{8}\ge\frac{3}{4}y\)\(\frac{z^3}{\left(x+1\right)\left(y+1\right)}+\frac{x+1}{8}+\frac{y+1}{8}\ge\frac{3}{4}z\)

Cộng theo vế của 3 BĐT trên, ta được: \(\frac{x^3}{\left(y+1\right)\left(z+1\right)}+\frac{y^3}{\left(z+1\right)\left(x+1\right)}+\frac{z^3}{\left(x+1\right)\left(y+1\right)}+\)\(\frac{x+y+z+3}{4}\ge\frac{3}{4}\left(x+y+z\right)\)

\(\Rightarrow\frac{x^3}{\left(y+1\right)\left(z+1\right)}+\frac{y^3}{\left(z+1\right)\left(x+1\right)}+\frac{z^3}{\left(x+1\right)\left(y+1\right)}\)\(\ge\frac{1}{2}\left(x+y+z\right)-\frac{3}{4}\ge\frac{1}{2}.3\sqrt[3]{xyz}-\frac{3}{4}=\frac{3}{2}-\frac{3}{4}=\frac{3}{4}\)

Vậy bất đẳng thức được chứng minh

Đẳng thức xảy ra khi x = y = z = 1 hay a = b = c = 1

Bình luận (0)
 Khách vãng lai đã xóa
Hoàng Diệu Thủy
Xem chi tiết
Trần Huy tâm
22 tháng 12 2019 lúc 19:20

áp dụng bất đẳng thức cauchy schwarz

\(\frac{1}{a+b}+\frac{1}{b+c}+\frac{1}{c+a}\ge\frac{\left(1+1+1\right)^2}{2\left(a+b+c\right)}=\frac{9}{2\cdot1}=\frac{9}{2}>4\)

suy ra điều phải chứng minh

Bình luận (0)
 Khách vãng lai đã xóa
Nguyen
27 tháng 1 2020 lúc 19:31

Cách 2:

VT=\(\frac{1}{1-c}+\frac{1}{1-b}+\frac{1}{1-a}\)\(\ge\frac{3}{\sqrt[3]{\left(1-a\right)\left(1-b\right)\left(1-c\right)}}\)

mà \(\sqrt[3]{\left(1-a\right)\left(1-b\right)\left(1-c\right)}\le\frac{3-\left(a+b+c\right)}{3}\)\(=\frac{2}{3}\)

=>\(VT\ge\frac{3}{\frac{2}{3}}=\frac{9}{2}>4\)

Bình luận (0)
 Khách vãng lai đã xóa
bt ko
Xem chi tiết
Võ Hồng Phúc
1 tháng 12 2019 lúc 13:07

sai đề

Bình luận (0)
 Khách vãng lai đã xóa